User avatar
 
ohthatpatrick
Thanks Received: 3807
Atticus Finch
Atticus Finch
 
Posts: 4661
Joined: April 01st, 2011
 
 
 

Q11 - Gyms and fitness centers are sometimes

by ohthatpatrick Tue Nov 05, 2019 3:30 pm

Question Type:
Principle-Strengthen

Stimulus Breakdown:
Conclusion: If you wanna buy a used exercise machine, a gym/fitness center can be a good place to buy it from.

Evidence: Although their stuff has been put through considerable use, it's really well made to begin with, it's well maintained, and it's sold for a reasonable price.

Answer Anticipation:
The three typical forms for correct answers on Principle-Strengthen are
1. IF premise, THEN conclusion
2. "Should" rule-of-thumb
3. Weigh the competing concerns

Since this argument has competing concerns, I would anticipate the answer will have a #3 feel. There's an upside (well made and well maintained and reasonable price) and a downside (been used a lot) to buying used machines from a gym. A correct answer might make us feel better about the tradeoff: "the quality of a used exercise machine has more to do with how well made/maintained it's been than how much it's been used."

Correct Answer:
C

Answer Choice Analysis:
(A) We're trying to conclude that a place is a good source for used exercise machines. This conditional starts "IF it's a good source ...". Any answer that says "IF conclusion" is always wrong.

(B) Cool, this looks good. It attaches really strong language "best" to the three qualities we know that gym machines often have.

(C) YES ... This looks even better. This gives us a conditional that takes what we know about the used machines at gyms and delivers us the language of the conclusion.

(D) This says "IF such and such, THEN you're NOT a good place to buy used machines." Any answer that says " ... THEN not-conclusion" is always wrong.

(E) This says "IF good source of used machines ...". Any answer that says "IF conclusion" is always wrong.

Takeaway/Pattern: The correct answers to Principle-Strengthen vary a good bit. Historically, they were usually If/Then conditionals that pretty much felt like Sufficient Assumption. That's what (C) is. In more recent times, there's been an uptick in correct answers that just strengthen, which is what (B) is doing. The question stem asks us which answer MOST helps, so picking something that basically guarantees the truth of the Conclusion, (C), is more powerful than picking something that makes you feel somewhat better about the competing tradeoffs of buying used from a gym, (B).

#officialexplanation
 
amr
Thanks Received: 0
Vinny Gambini
Vinny Gambini
 
Posts: 3
Joined: August 05th, 2020
 
 
 

Re: Q11 - Gyms and fitness centers are sometimes

by amr Wed Aug 05, 2020 7:50 pm

Patrick, shouldn't B have been eliminated immediately due to it stating the best machine is designed for home use? This contradicts the premise of the stimulus. Thus, C is not a stronger answer because it is a conditional, rather, B is just factually incorrect.
 
Laura Damone
Thanks Received: 94
Atticus Finch
Atticus Finch
 
Posts: 468
Joined: February 17th, 2011
 
This post thanked 1 time.
 
 

Re: Q11 - Gyms and fitness centers are sometimes

by Laura Damone Mon Aug 10, 2020 5:27 pm

I totally agree, AMR. I'd kill B right away because it focuses on machines designed for home use!
Laura Damone
LSAT Content & Curriculum Lead | Manhattan Prep